Board logo

标题: [讨论]请教一道逻辑题 [打印本页]

作者: lanshichao90    时间: 2009-5-21 20:58     标题: [讨论]请教一道逻辑题

A drug that is highly effective in treating many types of infection can, at present, be obtained only from the bark of the ibora, a tree that is quite rare in the wild. It takes the bark of 5,000 trees to make one kilogram of the drug. It follows, therefore, that continued production of the drug must inevitably lead to the ibora’s extinction.

Which of the following, if true, most seriously weakens the argument above?

(A) The drug made from ibora bark is dispensed to doctors from a central authority.

(B) The drug made from ibora bark is expensive to produce.

(C) The leaves of the ibora are used in a number of medical products.

(D) The ibora can be propagated from cuttings and grown under cultivation.E

(E) The ibora generally grows in largely inaccessible places.


D和E我觉得都成,一个说多种,一个说砍不到.......

作者: Redshoe08    时间: 2009-5-22 06:33

S                   N 

Continued production----->Extinction

A->B要削弱,应该A--->~B,(D)即A-->~B,而(E)实则~A-->~B

所以选D不选E 


作者: littleblues    时间: 2009-5-23 08:06

E不对!E是反对前提.

E说生长在无法接近的地方就是再说没法进行continued production of the drug .

削弱的时候是要承认前提反对结论(feifei的话),所以你这个不对!


作者: littleblues    时间: 2009-5-23 21:03

用楼上的话说,就是~A-->~B并不对A->B这个推理的合理性构成威胁.
作者: lanshichao90    时间: 2009-5-24 19:21

多谢!!!!!




欢迎光临 国际顶尖MBA申请交流平台--TOPWAY MBA (http://forum.topway.org/) Powered by Discuz! 7.2